Finance Test 2

Lakukan tugas rumah & ujian kamu dengan baik sekarang menggunakan Quizwiz!

You have the opportunity to purchase mineral rights to a property in Wyoming with expected annual cash flows of $8,000 per year for ten years. If you discount these cash flows at a rate of 12% per year, what are these cash flows worth today if the cash flows occur at the end of each period?

45,201.78 MODE = END INPUT 1012 ? -8,0000 KEY N I/Y PV PMT FV CPT 45,201.78

To determine the present value of a future amount, one should ________ the future cash flows.

discount

To determine the interest paid each compounding period, we take the advertised annual percentage rate and simply divide it by the ________ to get the appropriate periodic interest rate.

number of compounding periods per year

A never-ending stream of equal periodic, end-of-the-period cash flows is called a/an ________.

perpetuity

When interest rates are stated or given for loan repayments, it is assumed that they are ________ unless specifically stated otherwise.

annual percentage rates

deposit money in a certificate of deposit (CD) at a bank

The bank is technically renting money from you with a promise to repay that money with interest.

In constructing a yield curve you place interest rates on the vertical axis, and risk on the horizontal axis.

False In constructing a yield curve you place interest rates on the vertical axis, and TIME TO MATURITY on the horizontal axis.

Which of the below is NOT a major component of interest rates?

A. Real rate B. Inflation premium C. Default premium D. Historical interest rates Correct Answer: D

Which of the following statements is FALSE?

A. The APR can be referred to as a promised annual percentage rate B. The period in which interest is applied or the frequency of times interest is added to an account each year is called the compounding period or compounding periods per year. C. Although an APR is quoted on an annual basis, interest can be paid monthly but never daily. D. Although an APR is quoted on an annual basis, interest can be paid quarterly. Correct Answer: C

________ is simply the interest earned in subsequent periods on the interest earned in prior periods.

Compound interest

Which of the following actions will INCREASE the present value of an investment?

Decrease the interest rate.

Which of the following is the CORRECT formula for calculating the future value?

FV = PV × (1 + r)n

Which of the following investments has a larger future value: A $100 investment earning 10% per year for 5 years or a $100 investment earning 5% per year for 10 years?

FV = PV ∗ (1 + r)n = $100 ∗ (1.10)5 = $161.05 and $100 ∗ (1.05)10 = $162.89 MODE = END INPUT 510-1000? KEYNI/YPV PMT FV CPT 161.05 MODE = END INPUT 105-1000? KEY NI/YPV PMT FV CPT 162.89

The most common shape for a yield curve is upward sloping.

True

Suppose you invest $1,000 today, compounded quarterly, with the annual interest rate of 8.00%. What is your investment worth in one year?

$1,082.43 With PV = $1,000, APR = 8.00%, C/Y = 4, periodic interest rate = r = 0.02, we begin by taking (1 + periodic rate) to the power of C/Y. Doing this gives: (1.02)4 = 1.08243. Multiplying this number by PV gives $1,082.43. Or using a financial calculator, Present Value = -1,000, PMT =0, Rate = 8, Periods = 4, Compounding = 4, solve for FV = $1,082.43.

You put 20% down on a home with a purchase price of $250,000. The down payment is thus $50,000, leaving a balance owed of $200,000. The bank will loan the remaining balance at 3.91% APR. You will make annual payments with a 30-year payment schedule. What is the annual annuity payment under this schedule?

$11,439.96 MODE = END INPUT 303.91 -200,000 ? 0 KEY N I/Y PV PMT FV CPT 11,439.96

You wish to make a substantial down payment on a lake cottage and you currently have $15,725 invested at an annual rate of 2.50%. How much money will be in the account in 3.5 years if it continues to earn at its present rate?

$17,144 FV = PV ∗ (1 + r)n = $15,725 ∗ (1.025)3.5 = $17,144 MODE = END INPUT3.52.50-15,7250? KEYNI/YPVPMTFV CPT17,144

The Fisher Effect involves which of the items?

Nominal rate, the real rate, and inflation

The ________ compensates the investor for the additional risk that the loan will not be repaid in full.

default premium

Suppose you postpone consumption and invest at 6% when inflation is 2%. What is the approximate real rate of your reward for saving?

4% We can see that an inflation rate of 2% is 4% less than our 6% investment rate. Thus, 4% is the real rate of your reward for saving.

You currently have $3,500 invested at an annual rate of 8%. How long will it take for this investment to grow to a value of $5,000?

4.63 years MODE = END INPUT 8 -3,500 0 5,000 KEY N I/Y PV PMT FV CPT 4.63

You have a choice between a lottery lump sum payout of $5,509,253.62 today or a series of twenty annual annuity payments of $500,000 each (first cash flow one year from today). At what discount rate are you indifferent between the two choices?

6.50% MODE = END INPUT 20 ? -5,509,253.62 500,000 0 KEY N I/Y PV PMT FV CPT 6.50

You currently have $67,000 in an interest-earning account. From this account, you wish to make 10 year-end payments of $8,500 each. What annual rate of return must you make on this account to meet your objective?

MODE = END INPUT 10 ? -67,000 8,500 0 KEY N I/Y PV PMT FV CPT 4.58

If you can earn 5.25% per year on your investments, how long will it take to double your money?

MODE = END INPUT ? 5.25 -102 KEY N I/Y PV PMT FV CPT 13.55

William wishes to save enough money to purchase a retirement lake cabin. He is willing to spend $200,000 for the cabin and he can save $20,000 per year and invest the money into an account earning 7.00% per year. If his investments come in the form of equal annual end-of-the-year cash flows and the first cash flow is in exactly one year, how long will it take him to save enough money to buy the lake cabin?

MODE = END INPUT ? 7 0 20,000 -200,000 KEY N I/Y PV PMT FV CPT 7.84

What is the EAR if the APR is 5% and compounding is quarterly?

Slightly above 5.09% Using the EAR formula, we get 5.0945%, or slightly above 5.09%. EAR = [(1 + APR/m)m] -1 = [(1 + .05/4)4] - 1 = 5.0945%.

An investment of $100 today is worth $116.64 at the end of two years if it earns an annual interest rate of 8%. How much interest is earned in the first year and how much in the second year of this investment?

The interest earned in year one is $8.00 and the interest earned in year two is $8.64. FV = PV ∗ (1 + r)n = $100 ∗ (1.08)1 - $100 = $8.00 and$108(1.08)1 - $108 = $8.64

Your grandmother places $13,000 into an account earning an interest rate of 7% per year. After 5 years the account will be valued at $18,233.17. Which of the following statements is CORRECT?

The principal is $13,000, the time period is 5 years, the future value is $18,233.17, and the interest rate is 7%. The $13,000 is the principal or present value, the interest rate is stated as 7%, the time period is identified as 5 years, and the future value is $18,233.17.

A home improvement firm has quoted a price of $14,700 to fix up Eric's backyard. Five years ago, Eric put $12,500 into a home improvement account that has earned an average of 4.75% per year. Does Eric have enough money in his account to pay for the backyard fix-up?

Yes; Eric has $15,764.50 in his home improvement account. FV = PV ∗ (1 + r)n = $12,500 ∗ (1.0475)5 = $764.50 MODE = END INPUT54.75-12,5000? KEYNI/YPVPMTFV CPT15,764.50

A series of equal periodic finite cash flows that occur at the beginning of the period are known as a/an ________.

annuity due

Assume that you are willing to postpone consumption today and buy a certificate of deposit (CD) at your local bank. Your reward for postponing consumption implies that at the end of the year ________.

you will be able to buy more goods or services

Your finance professor suggests that you should have $3,000,000 in your retirement portfolio before you even THINK about retiring. Recently, your aunt sold valuable California real estate and handed you a check for $387,500. (This is the amount you have after paying taxes. She is now your favorite aunt.) How much of the $387,500 must you set aside today if you invest a portion of the money at an annual rate of 7.5% and you wish to retire in 38 years with the amount suggested by your finance professor?

$192,132 MODE = END INPUT 387.5 ? 0 -3,000,000 KEY N I/Y PV PM TFV CPT 192,131.96

You intend to buy a vacation home in eight years and plan to have saved $75,000 for a down payment. How much money would you have to place today into an investment that earns 9% per year to have enough for your desired down payment?

$37,640 MODE = END INPUT 89 ? 0 -75,000 KEY N I/Y PV PMT FV CPT 37,639.97

Your employer has agreed to place year-end deposits of $1,000, $2,000 and $3,000 into your retirement account. The $1,000 deposit will be one year from today, the $2,000 deposit two years from today, and the $3,000 deposit three years from today. If your account earns 5% per year, how much money will you have in the account at the end of year three when the last deposit is made?

$6,202.50 FV = Σ PV × (1 + r)n = $1,000 × (1.05)2 + $2,000 × (1.05)1 + $3,000 × (1.05)0 = $6,202.50.

You dream of endowing a chair in finance at the local university that will provide a salary of $250,000 per year forever, with the first cash flow to be one year from today. If the university promises to invest the money at a rate of 4% per year, how much money must you give the university today to make your dream a reality?

$6,250,000 PV = PMT/r = $250,000/.04 = $6,250,000.

Your department at work places $8,000 every year-end into an account earning 5%. The money is used when the corporate office fails to fully finance your profitable projects. The money has not been touched since the first deposit was made exactly six years ago. If the most recent deposit was made today, how much money is currently in the account?

$65,136.07 MODE = END INPUT 750 -8,000? KEY N I/Y PV PMT FV CPT 65,136.07

You have just won the Publisher's Clearinghouse lottery of $50,000 per year for twenty years, with the first payment today followed by nineteen more start-of-the-year cash flows. At an interest rate of 4%, what is the present value of your winnings?

$706,696.97 MODE = BEGIN INPUT 20 4 ? -50,0000 KEY N I/Y PV PMT FV CPT 706,696.97

Which of the statements below is TRUE?

A. The frequency of bankruptcy for a high-tech up-start firm is lower than for a blue-chip firm, so we see lower borrowing rates for start-ups than for mature firms. B. The frequency of bankruptcy for a high-tech up-start firm is higher than for a blue-chip firm, so we see lower borrowing rates for start-ups than for mature firms. C. The frequency of bankruptcy for a high-tech up-start firm is lower than for a blue-chip firm, so we see higher borrowing rates for start-ups than for mature firms. D. The frequency of bankruptcy for a high-tech up-start firm is higher than for a blue-chip firm, so we see higher borrowing rates for start-ups than for mature firms. Correct Answer: D

Which of the statements below is FALSE?

A. The prices of goods and services tend to decrease over time because of inflation. B. The real interest rate is the reward for waiting. C. The reward for postponing consumption implies that at the end of the year you will be able to buy more goods. D. Nominal interest rates are the sum of two major components: the real interest rate and expected inflation. Correct Answer: A


Set pelajaran terkait

By design Science Ch.2 how are cells alike and different

View Set

Physiological and Behavioral Responses of the Neonate

View Set

Sacraments and Morality Fall Chapter 7 Study Guide

View Set

Chapter 17: Legal Issues: Recognizing Your Small Business NeedsAssignment

View Set

Georgia Laws and Rules Pertinent

View Set